I'll give brainliest!

I'll Give Brainliest!

Answers

Answer 1

The square roots would not be between 9 and 10 is [tex]\sqrt{80}[/tex].

[tex]\sqrt[]{80}=8.944\\ \sqrt[]{90}=9.486\\ \sqrt[]{95}=9.746\\ \sqrt[]{88}=9.380[/tex]

What are square roots?

The opposite of squaring an integer is finding its square root. The result of multiplying a number by itself yields its square value, whereas the square root of a number may be found by looking for a number that, when squared, yields the original value. It follows that a, a = b if "a" is the square root of "b." Every integer has two square roots, one of a positive value and one of a negative value because the square of any number is always a positive number. For instance, the square roots of 4 are both 2 and -2. However, the square root of a number is typically only expressed as the positive value.

The value of a number's power 1/2 is the number's square root. It is the number whose product by itself yields the original number, to put it another way. It is symbolized by the character "." The number behind the square root sign is referred to as the radicand, whereas the square root symbol itself is known as a radical.

Learn more about square root visit: https://brainly.com/question/3120622

#SPJ1


Related Questions

Given the following table of values for f(x), find f(0).
x −3 −1 0 3 6 9
f(x) 1 8 1 0 13 −3

Answers

The value of the function at x = 0 will be 1.

What is a function?

A statement, principle, or rule that creates a relationship between two variables is known as a function. Functions are abundant in mathematics and are required for the creation of complex relationships.

When the required parameters and natural laws are given values, the expression yields the calculation result that the mathematical model represents.

The table represents a function.

x     f(x)

-3     1

-1     8

0      1

3     0

6     13

9     -3

In the above table, we can see that at the value of x = 0 in the function the value of the whole function is 1.

Let's learn more about the function

brainly.com/question/5245372

Solve for x. Then find m (8x+4)°
(10x-6)°
Both lines are intersecting and the two equations are vertical pairs

Answers

For the vertical angles, the value of x is found as 5. The measure of the angle ∠QRT = 44° for the two intersecting lines.

What is referred as the vertical angles?When two lines intersect at a point, vertical angles are formed. They are always on equal footing. In other words, four angles are formed anytime two lines pass or intersect. We can see that two opposite angles are equal, and these are referred to as vertical angles.

For the given pair of angles in the question.

Two lines are intersecting to form two equations are vertical pairs.

∠QRT =  ∠VRS (vertical angles)

Put the values.

8x + 4 = 10x - 6

Simplifying.

2x = 10

x = 5

Put the values of 'x' in the angle.

∠QRT = 8x + 4

∠QRT = 8×5 + 4

∠QRT = 44

Thus, the measure of the angles ∠QRT is found as 44°.

To know more about the vertical angles, here

https://brainly.com/question/12162963

#SPJ10

Solve the compound inequality -8≤x+4<5

Answers

Answer:

-12≤x<1

Step-by-step explanation:minus both side by 4

Part A answer and explanation pls​

Answers

Answer:

[tex]\frac{3}{13}[/tex]

Step-by-step explanation:

the fraction is formed by the number who prefer historical movies to the total preferring all movies.

total = 24 + 18 + 6 + 30 = 78

number preferring historical movies = 18

then fraction preferring historical movies is

[tex]\frac{18}{78}[/tex] ( divide numerator/ denominator by 6 )

= [tex]\frac{3}{13}[/tex] ← in simplest form

Answer:

From the Graph the total number of students who preferred historical movies =  18.

Similarly total number of  students = 24+18+6+30

                                                         = 78

Fraction of student who prefer Historical movies = 18/78

                                                                                = 3/13.

Which set of measurement could be the interior angle measures of triangle.A. 10°, 10°, 160°B. 15°, 75°, 90°C. 20°, 80°, 100°D. 35°, 35°, 105°E. 60°, 60°,60°

Answers

The sum of the interior angles of a triangle must add 180°.

C : 20 + 80 + 100 = 200 NO

D : 35+35+105 = 175 NO

A : 10+ 10+160 = 180 Yes

B : 15 +75 + 90 = 180 Yes

E :60+60+60 = 180 Yes

Growing linearly, the balance owed on your credit card doubles from $600 to $1200 in 6 months. If the balance were growing according to the exponential function f(x)=600(1+0.1220)^x where x represents the number of months, what would the balance be after 6 months? Round your answer to the nearest cent.

Answers

Evaluate the given expression at x=6. This is, replace every x in the equation for 6 and solve:

[tex]\begin{gathered} f(6)=600(1+0.1220)^6 \\ f(6)=600(1.1220)^6 \\ f(6)=600\cdot(1.99506) \\ f(6)=1197.04 \end{gathered}[/tex]

The balance after 6 months is 1197.04.

help me I'm practicing

Answers

we calculate the area of one trangle and multiply by 2

we apply formula of the triangle area

[tex]A=\frac{b\times h}{2}[/tex]

where b is the base and h the height of the triangle

then replacing

[tex]\begin{gathered} A=\frac{6\times5.2}{2} \\ \\ A=15.6 \end{gathered}[/tex]

area of one triangle is 15.6square centimeters

Area of both triangles

[tex]\begin{gathered} 15.6\times2 \\ =31.2 \end{gathered}[/tex]

Area of the two triangle bases is 31.2 square centimeters

(05.05 MC)A food truck did a daily survey of customers to find their food preferences. The data is partially entered in the frequency table. Complete thetable to analyze the data and answer the questions:Likes hamburgersboes not like hamburgers TotalLikes burritos41Does not like burritos54135Total110205Part A: What percentage of the survey respondents do not like both hamburgers and burritos? (2 points)Part & What is the marginal relative frequency of all customers that like hamburgers? (3 points)Part C Use the conditional relative frequencies to determine which data point has strongest association of its two factors. Use completesentences to explain your answer. (5 points)

Answers

EXPLANATION:

Given;

We are given a frequency table which displays a survey of numbers of customers that like hamburgers and burritos and those that do not like hamburgers and burritos.

Required;

We are required to analyze the table and use the values to answer the questions that follow.

Solution;

We shall begin by completing the table as follows;

Part A:

What percentage of the survey respondents do not like both burritos and hamburgers?

The percentage that do not like hamburgers is

[tex]Does\text{ }not\text{ }like\text{ }hamburgers=\frac{95}{205}[/tex]

The percentage that do not like burritos is

[tex]Does\text{ }not\text{ }like\text{ }burritos=\frac{135}{205}[/tex]

The percentage that does not like both burritos and hamburgers is;

[tex]\frac{54}{205}=0.263414634146[/tex]

Expressed as a percentage, this is

[tex]\begin{gathered} Percentage=0.263414634146\times100 \\ \\ Percentage=26.3414634146 \\ \\ Percentage=26.34\%\text{ }(rounded\text{ }to\text{ }the\text{ }nearest\text{ }hundredth) \end{gathered}[/tex]

The marginal relative frequency of all customers that like hamburgers is the total of all hamburger lovers divided by the total of all respondents.

[tex]\begin{gathered} Marginal\text{ }relative\text{ }frequency=\frac{hamburger\text{ }lovers}{total\text{ }respondents}=\frac{110}{205} \\ \\ Marginal\text{ }relative\text{ }frequency=0.536585365854 \\ \\ Marginal\text{ }relative\text{ }frequency=53.66\%\text{ }(rounded\text{ }to\text{ }2\text{ }decimal\text{ }places) \end{gathered}[/tex]

ANSWER:

Part (a) 26.34%

Part (b) 53.66%

In how many ways can 3person study groups beselected from a class of 25students?Note: nrn!r!(n-r)!nEnter

Answers

Answer:

2,300

Explanation:

This is given as:

[tex]\begin{gathered} 25\text{ combination 3 represented as:} \\ ^nC_r=\frac{n!}{r!(n-r)!} \\ n=25 \\ r=3 \\ ^{25}C_3=\frac{25!}{3!(25-3)!} \\ ^{25}C_3=\frac{25\times24\times23\times22!}{3!\times22!} \\ ^{25}C_3=\frac{25\times24\times23}{3\times2\times1} \\ ^{25}C_3=2300 \end{gathered}[/tex]

Therefore, there are 2,300 ways that 3 persons can be selected from 25 people

find the percent of change. Round to the nearest whole percent original:26 new:30

Answers

We want to find the percentage change;

We can do that using the formula;

[tex]\text{ \%P}=\frac{New-Original}{\text{Original}}\times100\text{\%}[/tex]

Given:

[tex]\begin{gathered} \text{Original = 26} \\ \text{New = 30} \end{gathered}[/tex]

Substituting the given values;

[tex]undefined[/tex]

Find the value of x.1714X = 93x = 93x= 3x = 485

Answers

Given a right angle triangle:

The hypotenuse of the triangle = 17

The legs of the triangle are the sides 14 and x

We will find x using the Pythagorean theorem as follows:

[tex]x^2+14^2=17^2[/tex]

Solve for x:

[tex]\begin{gathered} x^2=17^2-14^2=93 \\ \\ x=\sqrt[]{93} \end{gathered}[/tex]

So, the answer is the first option

[tex]x=\sqrt[]{93}[/tex]

How do I simplify radicals in simplest radical form?

Answers

Expressing in simplest radical form just means simplifying a radical so that there are no more square roots, cube roots e.t.c

Examples of radicals are

[tex]\sqrt[]{4},\sqrt[2]{8},\sqrt[3]{16}[/tex]

For exmaple,

Given the radical

[tex]\sqrt[]{12}[/tex]

To simplify into the simplest radical,

Factorize the perfect square

[tex]\sqrt[]{12}=\sqrt[]{4\times3}[/tex]

Then we take out the pairs

[tex]\begin{gathered} \sqrt[]{12}=\sqrt[]{4}\times\sqrt[]{3} \\ \sqrt[]{12}=\sqrt[]{2\times2}\times\sqrt[]{3} \\ \sqrt[]{12}=\sqrt[]{2^2}\times\sqrt[]{3} \end{gathered}[/tex]

Simplify the result

[tex]\begin{gathered} \text{Where} \\ \sqrt[]{2^2}=2 \\ \sqrt[]{12}=2\times\sqrt[]{3} \\ \sqrt[]{12}=2\sqrt[]{3} \end{gathered}[/tex]

Hence, the simplified radical of the example used is

[tex]\sqrt[]{12}=2\sqrt[]{3}[/tex]

As assistant manager of a soccer specialty store you have been asked to review the inventory costs associated with the store's best selling soccer shoe. The information you have available concerning this shoe follows: Average Demand =18 pairs per week; Standard Deviation of demand = 6 pairs Lead time for ordering shoes= 1 week; Ordering cost = $32 per order Cost of a pair = $90; Carrying cost per pair per year = 10% of cost of per pair Service level =98%, Z =2.05; The store operates 50 weeks per year.

Answers

The annual demand when the manager reviews the stock is 900 units.

What is the annual demand?

Mean demand, d = 18 units per week

Standard deviation of weekly demand, σd = 6 units per week

Lead Time, LT = 1 week

Order Cost, S = $32 per order

Unit Cost, C = $90 per unit

Holding Cost, H = 10% of C = $9 per unit per year

Service level, SL = 98%

z score = NORMSINV(SL) = NORMSINV(98%) = 2.05

Operating weeks = 50 weeks per year

The annual Demand, D will be:

= 50*18

= 900 units

Learn more about demand on

https://brainly.com/question/1245771

#SPJ1

Type the correct answer in each box.If matrix C represents (A − B) + A, the value of the entry represented by c41 is and the corresponding entry in (A + B) − A is .

Answers

Given the matrices A and B:

[tex]\begin{gathered} A=\begin{bmatrix}{-5} & {3} & {8} & {} \\ {3} & {6} & {-5} & {} \\ {5} & {-9} & {0} & {} \\ {7} & {3} & {4} & {}\end{bmatrix} \\ \\ B=\begin{bmatrix}{-7} & {-8} & {-5} & {} \\ {7} & {9} & {2} & {} \\ {2} & {5} & {-7} & {} \\ {2} & {8} & {-7} & {}\end{bmatrix} \end{gathered}[/tex]

We know that:

[tex]C=(A-B)+A=2A-B[/tex]

Then, using the matrices A and B:

[tex]\begin{gathered} C=2\cdot\begin{bmatrix}{-5} & {3} & {8} & {} \\ {3} & {6} & {-5} & {} \\ {5} & {-9} & {0} & {} \\ {7} & {3} & {4} & {}\end{bmatrix}-\begin{bmatrix}{-7} & {-8} & {-5} & {} \\ {7} & {9} & {2} & {} \\ {2} & {5} & {-7} & {} \\ {2} & {8} & {-7} & {}\end{bmatrix} \\ \\ C=\begin{bmatrix}{-10} & {6} & {16} & {} \\ 6 & {12} & {-10} & {} \\ 10 & {-18} & {0} & {} \\ 14 & 6 & 8 & {}\end{bmatrix}+\begin{bmatrix}{7} & {8} & {5} & {} \\ -{7} & -{9} & -{2} & {} \\ -{2} & {-5} & {7} & {} \\ {-2} & -{8} & {7} & {}\end{bmatrix} \\ \\ \therefore C=\begin{bmatrix}{-3} & 14 & 21 & {} \\ -1 & {3} & {-12} & {} \\ 8 & {-23} & 7 & {} \\ 12 & -2 & 15 & {}\end{bmatrix} \end{gathered}[/tex]

And the element C₄₁ (fourth row and first column) is:

[tex]C_{41}=12[/tex]

Now, for the matrix (A + B) - A = B:

[tex]B_{41}=2[/tex]

simplify (3^-2)^4 A 1/3^2B 3^2C 1/3^8D 3^8

Answers

Ok, so

We're going to simplify the following expression:

[tex](3^{-2})^4[/tex]

Remember that if we have two exponents elevating each other in the same base, they multiply.

So, this is equivalent to write:

[tex](3^{-2})^4=(3)^{-8}[/tex]

Now, we can rewrite the last expression in this new one:

[tex](3^{-2})^4=(3)^{-8}=\frac{1}{3^8}[/tex]


John and Ariana bought school supplies. John spent $10.30 on 5 notebooks and 7 pens. Ariana spent $7.20 on 4 notebooks and 4 pens. What is the cost of 1
notebook and what is the cost of 1 pen?

Answers

Answer:

$ 0.75 for one pen

$1.05 per notebook

Step-by-step explanation:

John and Ariana bought school supplies. John spent $10.30 on 5 notebooks and 7 pens. Ariana spent $7.20 on 4 notebooks and 4 pens. What is the cost of 1 notebook and what is the cost of 1 pen?

John: 5n + 7p = 10.5

let's solve for n:

5n + 7p = 10.5

subtract 7p from both sides:

5n + 7p -7p = 10.5 - 7p

5n = 10.5 - 7p

divide both sides by 5:

5n/5 = (10.5 - 7p)/5

n = 2.1 - 7/5 p

Ariana: 4n + 4p = 7.20

Let's substitute in: n = 2.1 - 7/5 p

4n + 4p = 7.20

4(2.1 - 7/5p) + 4p = 7.20

multiply left side:

8.4 - 5.6p + 4p = 7.2

subtract 8.4 from both sides:

8.4 - 5.6p + 4p - 8.4 = 7.2 - 8.4

- 5.6p + 4p = -1.2

combine p terms on left side:

-1.6p = -1.2

divide both sides by -1.6:

-1.6p/(-1.6) = -1.2/(-1.6)

p = 0.75 cents for one pen

Now solve to find the price of a notebook:

4n + 4p = 7.20 when p = 0.75

4n + 4(0.75) = 7.20

4n + 3 = 7.20

subtract 3 from both sides:

4n + 3 - 3 = 7.20 - 3

4n = 4.2

divide both sides by 4:

4n/4 = 4.2/4

n = 1.05 per notebook

CHECK: when n = 1,05 and p = 0.75

John: 5n + 7p = 10.5

5(1.05) + 7(0.75) = 10.5

10.5 = 10.5

Ariana: 4n + 4p = 7.20

4n + 4p = 7.20

4(1.05) + 4(0.75) = 7.20

7.2 = 7.2

Answer assumes no additional sales tax.

A toy rocket is shot vertically into the air from a launching pad 5 feet above the ground with an initial velocity of 32 feet per second. The height h, in feet, of the rocket above the ground at t seconds after launch is given by the function

h(t)=-16t²2 +32t+5.

How long will it take the rocket to reach its maximum height? What is the maximum height?

Answers

It takes the rocket to reach its maximum height h max = 17(ft).

The object's maximum height is the highest vertical position along its trajectory.

How to find a maximum height?The object's maximum height is the highest vertical position along its trajectory. Before reaching the highest point, the object is flying upwards and then falls. It means that the vertical velocity is equal to 0 at the highest point of projectile motion (v y = 0 v y = 0 v_y=0).

So, h(t) = -16+ 32t+5:

t max = time for maximum heightt max = 32 / 2*(-16)= 32 / 32 = 1h max = the maximum height above the groundh  max = h(1) = -16() + 32*1 +5-16+32+5 =  -16+ 37 = 21

Then, h max rocket = the maximum height of the toy rocket

h max rocket  = 21 -5 = 17(ft)t max = 1 secondh max = 17(ft)

Therefore, it takes the rocket to reach its maximum height h max = 17(ft)

The object's maximum height is the highest vertical position along its trajectory.

To learn more about maximum height, refer to:

brainly.com/question/12446886

#SPJ13

Solve 1/3x + 4/9 = 7/9

Answers

We want to solve

[tex]\frac{1}{3}x+\frac{4}{9}=\frac{7}{9}[/tex]

We can subtract 4/9 on both sides.

[tex]\begin{gathered} \frac{1}{3}x+\frac{4}{9}-\frac{4}{9}=\frac{7}{9}-\frac{4}{9} \\ \\ \frac{1}{3}x=\frac{7}{9}-\frac{4}{9} \\ \\ \frac{1}{3}x=\frac{7-4}{9} \\ \\ \frac{1}{3}x=\frac{3}{9} \end{gathered}[/tex]

Now we have 3/9 on the right side, but we can simplify that fraction to 1/3

[tex]\begin{gathered} \frac{1}{3}x=\frac{3}{9} \\ \\ \frac{1}{3}x=\frac{1}{3} \end{gathered}[/tex]

And now we have the result!

[tex]x=1[/tex]

Therefore the final result is x = 1

Select the postulate that is illustrated for the real numbers.

25 + 0 = 25
A The commutative postulate for multiplication
B The multiplication inverse
C The addition inverse postulate
D Commutative postulate for addition
E The distributive postulate
F Additive identity
G Multiplication by one

Answers

Answer:

Additive Identity

Step-by-step explanation:

Hello!

Adding 0 to any number will give you that number itself, as proven by adding 0 to 25, which gave us 25.

This is the identity property of addition, as the output would be identical to the sum of the non-zero terms.

We can also experiment with these:

21 + 4 + 0 = 21 + 4 = 257(3 + 0) = 7(3) = 21

A number cube with faces labeled 1 to 6 is rolled once.
The number rolled will be recorded as the outcome.
Consider the following events.
Event A: The number rolled is less than 5.
Event B: The number rolled is even.
Give the outcomes for each of the following events.
If there is more than one element in the set, separate them with commas.
(a) Event "A or B": {0}
(b) Event "A and B": {0}
(c) The complement of the event A

Answers

According to the solving the probability are as follows:

a) Event "A or B": {1, 2, 3, 4, 6}

(b) Event "A and B": {2, 4}

(c) The complement of the event A: {5, 6}

Define the experiment's sample space and event:

The term "sample space" refers to a collection of probable results from a random experiment. The letter "S" indicates that this is the sample space. Events are a subset of what might happen in an experiment. Depending on the experiment, the outcomes in a sample area could change. In discrete or finite sample spaces, there are only a finite number of possible outcomes.

What is probability?

Simply put, probability measures how probable something is to occur. We can discuss the probabilities of various outcomes, or how likely they are, whenever we are unsure of how an event will turn out. Statistics is the study of events subject to probability.

According to the given data:

the sample space is, 1, 2, 3, 4, 5, 6 i.e., U = {1, 2, 3, 4, 5, 6}

Event A: The number rolled is less than 5

i.e., A = {1, 2, 3, 4}

Event B: The number rolled is even

i.e., B = {2, 4, 6}

Event A Or B A∪B = {1, 2, 3, 4} ∪ {2, 4, 6}

= {1, 2, 3, 4, 6}

Event "A and B"A∩B = {1, 2, 3, 4} ∩ {2, 4, 6}

= {2, 4}

Compliment of the event A

Comp(A) = U/ A = {1, 2, 3, 4, 5, 6} / {1, 2, 3, 4}

= {5, 6}

To learn more about events in probability visit:

https://brainly.com/question/11234923

#SPJ13

Is (x+1) a factor of -2x^5-4x^4+x-10?

Answers

Notice that:

[tex]-2x^5-4x^4+x-10,[/tex]

cannot be expressed as a product of factors, meaning that

[tex](x+1),[/tex]

is not a factor of the given polynomial.

Answer: Not a factor.

If f(x) = x² + 5 and g(x) = 3x, find (f o g)(x) and (g o f)(x) .

Answers

Answer:

[tex](f \circ g)(x) = 9x^2 + 5\\\\\\(g \circ f)(x) = 3x^2 + 15[/tex]

Step-by-step explanation:

We are given
[tex]f(x) = x^2+5\\\\g(x) = 3x\\\\(f\circ g)(x) = f(g(x))\\\\[/tex]

To find this, wherever you see an x in f(x) substitute the expression in g(x)

[tex](f\circ g)(x) = f(g(x))\\\\= f(3x)\\\\= (3x)^2 + 5\\\\=9x^2 + 5\\\\\\[/tex]

To find [tex](g \circ f)(x) = g(f(x))\\\\[/tex]

Wherever there is an x in the expression for g(x) substitute that x with the expression in f(x)

[tex](g \circ f)(x) = g(f(x))\\\\\\= g(x^2 + 5) = 3(x^2 + 5) \\\\= 3x^2 + 15[/tex]

R= {y | y is an integer and -5 ≤ y ≤ -4}

Answers

The R= {y | y is an integer and -5 ≤ y ≤ -4} is null set because there is no integers between -5 and -4.

The set is

R= {y | y is an integer and -5 ≤ y ≤ -4}

An integer is a combination of number zero, all the positive numbers and negative numbers without any fraction.

Any set that does not contains any numbers or elements is called null set, which is also called empty set or void set. Null set can be denoted as {} or ∅.

The set is R= {y | y is an integer and -5 ≤ y ≤ -4}

There is no integers between -5 and -4, therefore it is a null set

Hence, The R= {y | y is an integer and -5 ≤ y ≤ -4} is null set because there is no integers between -5 and -4.

The complete question is :

Write the set in roaster form R= {y | y is an integer and -5 ≤ y ≤ -4}

Learn more about null set here

brainly.com/question/22057145

#SPJ1

Triangle ABC is dilated by a scale factor of 4 to form triangle A’B’C

The coordinates of Vertex A’ are

The Coordinates of Vertex B’ are

The coordinates of Vertex C’ are

PLS HELP!!

Answers

The coordinates of vertex A is -2,1
The coordinates of vertex B is -3,3
The coordinates of vertex C is -1,3
my apologies if im incorrect but i think this the answer.

HELP ME!!!!!
One interior angle of a triangle is 95.5°, and the other two interior angles are congruent. What is the degree measure of one of the congruent angles?

42.25°
47.75°
84.5°
90°

Answers

The sum of all interior angles of a triangle is 180° thus the measure of one of the congruent angles is 42.25° so option (A) is correct.

What is a triangle?

A triangle is a 3-sided shape that is occasionally referred to as a triangle. There are 3 sides and three angles in every triangle, some of which may be the same.

Let's suppose the two congruent interior angles are x degrees.

It is known that the sum of all three angles inside a triangle will be 180°.

So,   m∠x+  m∠x +  95.5° = 180°

2m∠x = 180° - 95.5°

m∠x = 84.5/2 = 42.25°

Hence "The sum of all interior angles of a triangle is 180° thus the measure of one of the congruent angles is 42.25°".

For more about triangles,

https://brainly.com/question/2773823

 

#SPJ1  

This graph shows the solutions to the inequalities y> 3x-2 and y<?x-10Does the system of inequalities have solutions? If so, which region containsthe solutions?108A84210-88441022 488BPopс-10A. There is a solution, and it is shown by region A.B. There is a solution, and it is shown by region C.C. There is no solution.D. There is a solution, and it is shown by region B.

Answers

Answer:

there is no solution (option C)

Explanation:

Given:

y > 3/2x - 2

y < 3/2x - 10

To find:

The solution to both graphs

To determine the solution of the graphs, we will consider their slopes

[tex]\begin{gathered} y\text{ > }\frac{3}{2}x\text{ - 2} \\ comparing\text{ with y = mx + b} \\ m\text{ = slope, b = y-intercept} \\ from\text{ the above, the slope = 3/2} \end{gathered}[/tex][tex]\begin{gathered} y\text{ < }\frac{3}{2}x\text{ - 10} \\ the\text{ slope = 3/2} \end{gathered}[/tex]

The slope of both inequalities is 3/2. If the slopes of two lines are the same, the lines are said to be parallel. This means both inequalities are parallel lines

For parallel lines, there is no solution because the lines do not intersect (meet).

Since both inequalities give parallel lines, there will be no solution (option C)

An invester invested a total of 3,300 in two mutual funds. One fund earned a 5% profit while the other earned a 2% profit. If the investor's total profit was $126, how much was invested in each mutual fund?

Answers

Given

Total investment of $3,300

5% profit and 2% profit totaling $126

[tex]\begin{gathered} \text{Let} \\ x\text{ be the investment on 5\% profit} \\ y\text{ be the investment on 2\% profit} \end{gathered}[/tex]

The equations therefore will be

[tex]\begin{gathered} x+y=3300\text{ based on the total amount of investment} \\ 0.05x+0.02y=126\text{ based on the investor's total profit} \end{gathered}[/tex]

Use substitution method using the first equation

[tex]\begin{gathered} x+y=3300 \\ y=3300-x \\ \\ \text{Substitute }y\text{ to the second equation} \\ 0.05x-0.02y=126 \\ 0.05x-0.02(3300-x)=126 \\ 0.05x-66-0.02x=126 \\ 0.05x-0.02x=126-66 \\ 0.03x=60 \\ \frac{0.03x}{0.03}=\frac{60}{0.03} \\ \frac{\cancel{0.03}x}{\cancel{0.03}}=\frac{60}{0.03} \\ x=2000 \end{gathered}[/tex]

Now that we have solve for x, substitute it to the first equation to get the value of y

[tex]\begin{gathered} x+y=3300 \\ 2000+y=3300 \\ y=3300-2000 \\ y=1300 \end{gathered}[/tex]

Therefore, the amount invested in mutual fund that earned 5% was $2000, and the amount invested that earned 2% was $1300.

When Ross solved the equation 12 = 3x, he made a mistake. Hiswork is shown below. What mistake did Ross make?3x = 123x - 3= 12-3x=9

Answers

The mistake Ross has made was in line 2, and it is that we can only combine like terms, that is, terms that have the same variable at the same power. So

[tex]3x-3\ne x,[/tex]

We cannot performe this operation.

3.)The area of a regular polygon is 145.8 sq. cm. If the perimeter of this polygon is 108 cm,find the length of the apothem.

Answers

The area of the regular polygon is a= 145.8 sq.cm

Perimeter of the polygon is p=108 cm.

Let apothem length be a

We know,

a=p x a/2

Putting the values,

145.8=108xa/2

a=2.7 cm

The length of the apothem is 2.7 cm

i need tutor answer this two quedtion thabks you so much

Answers

5. Let the following inequality:

[tex]13-3m\text{ }<-2[/tex]

this is equivalent to:

[tex]-13+3m\text{ }>2[/tex]

this is equivalent to:

[tex]-13+13+3m\text{ }>2\text{ +13}[/tex]

this is equivalent to:

[tex]3m\text{ }>15[/tex]

solve for m:

[tex]m\text{ }>\frac{15}{3}=5[/tex]

that is :

[tex]m\text{ }>5[/tex]

6. Because of the graphic (real line), we can conclude that the correct interval would be:

[tex](-\infty,-3\rbrack\text{ = x}\leq-3[/tex]

Then we have to find the inequalities that have this solution interval.

a) Let the inequality

[tex]\frac{x}{3}+2\leq1[/tex]

this is equivalent to:

[tex]\frac{x}{3}+2-2\leq1-2[/tex]

this is equivalent to:

[tex]\frac{x}{3}\leq-1[/tex]

solve for x:

[tex]x\leq-3[/tex]

then. A) represent the graph.

b)Let the inequality

[tex]8-5x\ge23[/tex]

this is equivalent to:

[tex]-8+5x\leq-23[/tex]

this is equivalent to:

[tex]-8+8+5x\leq-23+8[/tex]

this is equivalent to:

[tex]5x\leq-15[/tex]

solve for x:

[tex]x\leq\frac{-15}{5}\text{ = -3}[/tex]

then the solution interval would be:

[tex]x\leq\text{-3}[/tex]

then. B) represent the graph.

c) Let the inequality:

[tex]-18\ge3+7x[/tex]

this is equivalent to:

[tex]-18+18\ge3+18+7x[/tex]

this is equivalent to:

[tex]0\ge21+7x[/tex]

this is equivalent to:

[tex]-21\ge7x[/tex]

solve for x:

[tex]x\text{ }\leq\frac{-21}{7}\text{ = -3}[/tex]

that is:

[tex]x\text{ }\leq\text{-3}[/tex]

then. C) represent the graph.

d) Let the inequality:

[tex]7x-3\leq18[/tex]

this is equivalent to:

[tex]7x-3+3\leq18+3[/tex]

this is equivalent to:

[tex]7x\leq21[/tex]

solve for x:

[tex]x\leq3[/tex]

We can conclude that this interval does not represent the graph because:

[tex]x\leq3\text{ }\ne\text{ }x\leq-3\text{ }[/tex]

Finally:

e) Let the inequality:

[tex]1-\frac{x}{2}\text{ }\leq2\text{ +}\frac{1}{2}[/tex]

this is equivalent to:

[tex]-1+\frac{x}{2}\text{ }\ge-2\text{ -}\frac{1}{2}[/tex]

this is equivalent to

[tex]-2+x\text{ }\ge-4\text{ -}1[/tex]

that is:

[tex]x\text{ }\ge-4\text{ -}1+2[/tex]

that is:

[tex]x\text{ }\ge-3[/tex]

THEN WE CAN CONCLUDE THAT THE CORRECT ANSWER ARE:

A), B), C) AND E)

Other Questions
Read the excerpt below.Katherine Johnson was born on August 26, 1918 and her genius was evident from a young age. She began high school when she was just ten years old and graduated from college in 1937 at the age of eighteen with two degrees. After college, Katherine became a teacher until she left the profession to go back to school for a master's degree in mathematics. She did not finish her degree. Instead, she got married and started a family.How does the author organize information in this passage to create a clear central idea? Cause and effect Chronological Compare and contrast Problem and solution what benefits does kipling say westerners bring to non-europeans? Ima Stomaksic is walking from lunch to her PE class. She exits the lunchroom and walks 43m west. She then turns and walks 72m north down the hallway leading to the locker room. Determine the magnitude and direction of Ima's resultant displacement, and the angle of the displacement. Find the direction of the sumof these two vectors:16.3 m7.70 m20.0magnitude (m)A27.8 lan and Matthew went shopping. lan spent $20 less than twice as much as Matthew spent. In total, they spent $130. What is 30 percent of 120? -27 = -w/5 solve for w simplify your answer as much as possible a national package delivery company relies heavily on a hierarchy culture to manage its vast and complex shipping processes. are part of a hierarchy culture when describing specialty channelers we mean that select one: a. the company specializes in special interest tours b. the tours are for people in special groups such as churches c. they are specialty intermediaries d. they include incentive travel firms, corporate travel offices e. c and d Read the excerpt below.Katherine Johnson was born on August 26, 1918 and her genius was evident from a young age. She began high school when she was just ten years old and graduated from college in 1937 at the age of eighteen with two degrees. After college, Katherine became a teacher until she left the profession to go back to school for a master's degree in mathematics. She did not finish her degree. Instead, she got married and started a family.How does the author organize information in this passage to create a clear central idea? Cause and effect Chronological Compare and contrast Problem and solution two friends went to a restaurant and ordered one plain pizza and two sodas. the bill totaled $15.95. later that day, five friends went to the same restaurant. they ordered 3 plain pizzas and 5 sodas. their bill totaled $45.90.write and solve a system of equations to determine the the price of one plain pizza(only an algebraic solution can receive full credit.) Two cheers for Democracy; one because it admits variety and two because it permits criticism.Please help ASAP 100 points+ brainlyest! I need this done today! I only need an opinion agreeing with this statement and evidence What inferences can you make about how each of the following images supports characterization and mood? the trees and the swamp (lines 4047) the hewn trees (lines 96-102) Toms new house (lines 270272) Tom as a churchgoer (lines 279289) Identify the reasoning that would justify each step in solving for m Someone help me please Refer to the poem the sea view Taking vitamin supplements can help----------- to your immune system. Why do you think early humans created art? Were the reasons for creatingart different from or similar to reasons for creating art today? which structure is highlighted? a rounded ridge, located on the posterior side of the femoral head runs aslant from the summit of the greater trochanter.which structure is highlighted? intertrochanteric crest linea aspera gluteal tuberosity lesser trochanter i need help asp ;-;Which of the following values is less than |58|?A. 48B. 68C. 68D. 48 Name the point that lies on this line: y = 2 + 5(x + 7)